PT11.S2.Q06 - cigarette advertisements

Waiting For Grey DayWaiting For Grey Day Alum Member
edited September 2017 in Logical Reasoning 323 karma

I think this question really gets at the subtly of the "support" role we have to play to make the relationship P -> C more relevant.

I oscillated between A and E but eventually went with A, primarily because I thought the phrase "shout not be allowed" matched nicely with the "government should ban". Even during BR, I stuck to my original choice, which tells me I clearly don't understand this question. I do see that (E) has a conditional statement Promote -> Healthy and that the Healthy is denied because of the premises given in the stimulus but I felt that was also similar for (A). (A) has "people doing things that endanger their health".

So my thought for (E) was that, even though it does suggest that "If not healthy, ad should not promote", it doesn't necessarily link the "government should ban" aspect. This is such a curve breaker question (and there is no JY explanation for this) that I was really hoping someone could shed some light on this question.

If possible, could anyone share their reasoning/understanding of this question?
Thanks in advance :)

Comments

  • LSATcantwinLSATcantwin Alum Member Sage
    edited September 2017 13286 karma

    I think A doesn't quite do it!

    If an advertisement is prevented from showing people doing things that endanger their health does that necessary stop advertisements from promoting smoking?

    What if the advertisement is a giant cartoon cigarette that tells you smoking is cool? That is an ad that does not show someone endangering their health and yet it still promotes smoking.

    E makes it much harder. Smoking is unhealthy and if we only allow advertisements to promote healthy things then smoking cannot be promoted in any way at all!

  • BinghamtonDaveBinghamtonDave Alum Member 🍌🍌
    8689 karma

    This is a very difficult question and one in which I feel I would get wrong 10 times out of 10. I'm not too certain of the usefulness of this question to be honest. I try never to say that, but I believe it is warranted in this case.

    We are asked to strengthen via the use of a principle.

    Our core is straightforward:
    P: smoking has been shown to be a health problem
    C: governments should ban all advertisements that promote it

    I dislike this question because I feel the test taker is put in a position to calculate the "reasonableness" of competing assumption(s) within two answer choices.

    A: Advertisers should not be allowed to show people doing things that are health hazards.
    To make this strengthen the argument, we must assume: 1."Not be allowed" means a government ban and 2."People doing" is a reasonable stand in for "ban all." More specifically, we have to assume that the "people showing" would encapsulate "all." I don't know how reasonable it would be to assume those two things to strengthen this argument.

    E: Might be even worse. For E we are told advertisers should promote only healthy products. We fail the necessary condition and are left with: "advertisers should not promote." But where does that get us? How does that at all get us to support "the government should ban all" that is quite the gap to be asked to be alright with in my estimation.

    As an aside, there is a massive lacuna between what one should not do and what the government should ban all of. For instance, one should not abuse alcohol, but nothing in that statement supports the contention that the government should ban all alcohol.

    Now, there has to be an answer that "most supports" the argument from these two choices. I personally am not sure on precisely what grounds we should go forth strengthening that argument given the available choices. I look forward to a correspondence on this question in this thread. There is always the possibility that I have misread or am not emphasizing the correct part of the problem.

    David

  • LSATcantwinLSATcantwin Alum Member Sage
    13286 karma

    @BinghamtonDave Look at my explanation above. I am wondering what you have to say about my line of thinking because it seemed fairly clear to me...

    A: Prevent from showing people doing things that endanger their health.

    What about cartoon ads or ad's that are purely text based that promote smoking? If our goal is to ban all advertisement that promotes smoking surely this does nothing. I've shown two examples where I can still promote smoking while not showing people smoke/harm their health.

    E: Advertisements only promote healthy products.

    Basically this makes up the lost ground. Smoking is inherently unhealthy. If I can only advertise things that are healthy then no ad that promotes smoking is okay.

    Or am I missing something?

  • Waiting For Grey DayWaiting For Grey Day Alum Member
    323 karma

    @LSATcantwin said:
    I think A doesn't quite do it!

    If an advertisement is prevented from showing people doing things that endanger their health does that necessary stop advertisements from promoting smoking?

    What if the advertisement is a giant cartoon cigarette that tells you smoking is cool? That is an ad that does not show someone endangering their health and yet it still promotes smoking.

    E makes it much harder. Smoking is unhealthy and if we only allow advertisements to promote healthy things then smoking cannot be promoted in any way at all!

    That's an excellent point. Now, I can see that (A)'s "people doing things" require us to assume that cigarette ads have people inhaling/exhaling cigarette smoke. I guess when it comes down to questions like this, you have to really be picky with any unwarranted assumptions. Thank you for your response!

  • Waiting For Grey DayWaiting For Grey Day Alum Member
    edited September 2017 323 karma

    @BinghamtonDave said:
    This is a very difficult question and one in which I feel I would get wrong 10 times out of 10. I'm not too certain of the usefulness of this question to be honest. I try never to say that, but I believe it is warranted in this case.

    We are asked to strengthen via the use of a principle.

    Our core is straightforward:
    P: smoking has been shown to be a health problem
    C: governments should ban all advertisements that promote it

    I dislike this question because I feel the test taker is put in a position to calculate the "reasonableness" of competing assumption(s) within two answer choices.

    A: Advertisers should not be allowed to show people doing things that are health hazards.
    To make this strengthen the argument, we must assume: 1."Not be allowed" means a government ban and 2."People doing" is a reasonable stand in for "ban all." More specifically, we have to assume that the "people showing" would encapsulate "all." I don't know how reasonable it would be to assume those two things to strengthen this argument.

    E: Might be even worse. For E we are told advertisers should promote only healthy products. We fail the necessary condition and are left with: "advertisers should not promote." But where does that get us? How does that at all get us to support "the government should ban all" that is quite the gap to be asked to be alright with in my estimation.

    As an aside, there is a massive lacuna between what one should not do and what the government should ban all of. For instance, one should not abuse alcohol, but nothing in that statement supports the contention that the government should ban all alcohol.

    Now, there has to be an answer that "most supports" the argument from these two choices. I personally am not sure on precisely what grounds we should go forth strengthening that argument given the available choices. I look forward to a correspondence on this question in this thread. There is always the possibility that I have misread or am not emphasizing the correct part of the problem.

    David

    @BinghamtonDave Thank you for your response! I find solace in knowing that you also consider this question very difficult :) As you mentioned, each answer choice requires from us certain assumptions but even then, their comparative level of support doesn't seem to match up to the conclusion. I also think this is partly because the argument is so simple but full of gaping holes.

    But I think your breakdown of assumptions required for (A) really revealed how (A) lends less support to the structure than (E). I agree that "should not be allowed" doesn't necessarily mean "government should ban". Lightbulb moment there.

    I took a closer look at the question with your response and @LSATcantwin 's response in mind and tried to find what the missing assumption is and how (E) exposes it and affirms it.

    P: Cigarette smoking is unhealthy
    C: Gov should ban all ads that promote smoking

    Missing assumption: Ads shouldn't promote smoking because it is unhealthy

    I think this is what (E) attempts to close the gap on and what (A) disguises to be doing as well. But like you mentioned, (A)'s "not be allowed to show people doing things" sets the scope of ads in a slightly different tone than the one we need for the argument.

  • BinghamtonDaveBinghamtonDave Alum Member 🍌🍌
    8689 karma

    @LSATcantwin thank you for the response
    (E) states: "Advertisements should promote---->healthy"
    Healthy---->Should promote
    who/what should not promote? Advertisements

    I believe the problem I see here is that this answer choice is telling us what advertisements should or should not do. Our conclusion is concerned with a specific action government should take. The fact that an advertisement does something that advertisements should not do is quite a leap away from government ban. Ideally, what I would like to see is:
    Governments should allow only ads for healthy products

    The growth in my own personal shortcoming with respect to this question should be as follows: sometimes I am going to get an answer choice that on other questions actually won't be the most effective at strengthening, in fact it might be something I dismiss quickly. In short, I might get an answer choice that would be weak on one question but the strongest of a bunch on another. The LSAT is mostly consistent: meaning there are underlying patterns one can get ahold of with LR. But every once in awhile there are answer choices that are simply the best of the bunch that aren't homeruns.

    I have found the following strategies useful on such questions:
    -The attractive wrong answer choice will be masquerading as something we hope it says.
    -The credited response might be something that we have dismissed previously

    David

Sign In or Register to comment.